Download as pdf or txt
Download as pdf or txt
You are on page 1of 57

1. On December 2015, Killua Ltd. acquired all the assets and liabilities of Gon Ltd. with Killua Ltd.

issuing
100,000 shares to acquire these net assets. The fair value of Gon Ltd.’s assets and liabilities at this date were:

Cash P50, 000


Furniture and Fittings 20, 000
Accounts Receivable 5, 000
Plant 125, 000
Accounts Payable 15, 000
Current Tax Liability 8, 000
Provision for annual leave 2, 000
The financial year for Killua Ltd. is January- December.
The fair value of each Killua Ltd. share at acquisition date is 1.90. At acquisition date, the acquirer could only
determine a provisional fair value for the plant. On March 1, 2016, Killua Ltd. received the final value from the
independent appraisal, the fair value at acquisition date being P131, 000. Assuming the plant had a further
five year life from the acquisition date.
The amount of goodwill arising from the business combination at December 1, 2015 ?
a. P15, 000 c. P5, 000
b. 9, 000 d. 0

ANSWER: B

Consideration transferred (100, 000 x 1.90) P190, 000


Less: Fair Value of net identifiable assets acquired
Cash P50, 000
Furniture & Fittings 20, 000
Accounts Receivable 5, 000
Plant 131, 000
Accounts Payable (15, 000)
Current tax liability (8, 000)
Liabilities (2, 000) 181, 000
Goodwill P9, 000

2. The E. Vendivel Company acquired the net assets of the Vivar Company on January 1, 2015 and made the
following entry to record the purchase:
Current Assets .................................... 100, 000
Equipment ........................................ 150, 000
Land ....................................................50, 000
Buildings ............................................300, 000
Goodwill ............................................ 100, 000
Liabilities…………………………………. 80, 000
Common Stock, P1 par……………. 100, 000
Paid-in capital in excess of par… 520, 000

Assuming that the additional shares on January 1, 2017 would be issued on that date to compensate for any
fall in the value of E. Vendivel common stock below P16 per share, the settlement would be to cure the
deficiency by issuing added shares based on their fair values on January 1, 2017. The fair price of the shares
on January 1, 2017 was P10.

What is the additional number of shares issued on January 1, 2017 to compensate for any fall in the value of
the stock?

a. P160, 000 c. 60, 000


b. 100, 0000 d. 10, 000
ANSWER: C
Deficiency (16-10) x 100, 000shares issued to acquire .............................. P600, 000
Divided by: Fair value of the share .................................................. 10
Additional number of shares to issued…………………………………………………. P60, 000

3. X Company acquires all of Y Company in an acquisition properly accounted for as an asset acquisition. X
issues 80,000 shares of common stock with a fair value of P8,000,000 for Y’s net assets. The fair values of
Y’s assets and liabilities approximate their book values, except Y has customer lists valued at P3,000,000 that
are not reported on its balance sheet, and its plant assets are overvalued by P5,000,000. Here are the
balance sheets of X and Y prior to the acquisition:

X Company Y Company

Assets P30,000,000 P10,000,000

Liabilities P16,000,000 P 6,000,000

Common stock, $1 par 1,000,000 100,000

Additional paid-in capital 9,000,000 2,900,000

Retained earnings 4,000,000 1,000,000

P30,000,000 P10,000,000

How much goodwill is recognized for this acquisition?

a. P 2,000,000
b. P 3,000,000
c. P 6,000,000
d. P 11,000,000

ANS: C

Cost P8,000,000
Fair value of net assets acquired
Reported assets P 5,000,000
Customer lists 3,000,000
Liabilities (6,000,000) 2,000,000
Goodwill P6,000,000

4 .P acquires all of the voting shares of S by issuing 500,000 shares of P1 par common stock valued
at P10,000,000. Included in the agreement is a contingency guaranteeing the former shareholders of
S that P's shares will be worth at least P18 per share after one year. If the shares are worth less, P will
pay the former shareholders of S enough cash to reimburse them for the decline in value below P18
per share. P estimates that there is a 5% chance that the stock value will be P16 at the end of one
year, and a 95% chance that the stock value will be P18 per share or higher. A discount rate of 10% is
appropriate. What is the value of the stock price contingency at the date of acquisition?

a. P 1,000,000
b. P 45,455
c. P 50,000
d. P 863,636

ANS: B

Rationale: [(P18 - P16) x 500,000] x .05 = P50,000/1.10 = P45,455

5. P purchased all of the outstanding shares of S for P1,300,000 at a time when the underlying book
value of S was P1,200,000. S's assets and liabilities consist of the following:

Fair value Book value

Cash, receivables P250,000 P250,000

Inventory 360,000 380,000

Equipment 900,000 600,000

Liabilities 30,000 30,000

The gain on acquisition is:

a. P140,000
b. P180,000
c. P220,000
d. P260,000
ANS: B

Rationale:

Cost P1,300,000

Book value 1,200,000

Excess of cost over book value P 100,000

Excess

Inventory P(20,000)

Equipment 300,000 280,000

Gain P 180,000

For questions 6-7:

P Company acquired all of the net assets of S Company. The balance sheet of S Company immediately prior
to the acquisition, along with market values of its assets and liabilities, is as follows:

Accounts S Company
book value market value
Current assets P 800,000 P 1,000,000
Plant & equipment (net) 28,000,000 35,000,000
Patents 100,000 2,000,000
Identifiable intangible: brand names 0 13,000,000
Skilled work force 0 4,000,000
Goodwill 200,000 700,000
Liabilities 21,000,000 20,000,000
Common stock, $10 par 2,000,000
Additional paid-in capital 3,000,000
Retained earnings 3,100,000

6. P Company pays P40,000,000 in cash for S Company, in an acquisition properly reported as a


statutory merger. P records goodwill of:

a. P18,000,000
b. P17,300,000
c. P 9,000,000
d. P 4,300,000

ANS: C
Rationale: P9,000,000 = P40,000,000 – (P1,000,000 + P35,000,000 + P2,000,000 + P13,000,000 -
P20,000,000).

7. Now assume P Company pays P30,000,000 in cash to acquire S Company, in an acquisition properly
reported as a statutory merger. P records a gain on acquisition of:

a. Zero
b. P1,000,000
c. P1,700,000
d. P 5,700,000

ANS: B
Rationale: P(1,000,000) = P30,000,000 – (P1,000,000 + P35,000,000 + P2,000,000 + P13,000,000 -
P20,000,000).

8. Bats Inc, a new corporation formed and organized because of the recent consolidation of II Inc, and JJ Inc.,
shall issue 10% participating preferred stocks with a par value of P100 for II and JJ net assets contribution,
and common shares with a par value of P50 for the difference between the total shares to be issued and the
preferred shared issued. The total shares to be issued by Bats shall be equivalent to average annual earnings
capitalized at 10%. Relevant data on II and JJ follows:

II JJ

Total assets P720,000 P921,600

Total liabilities 432,000 345,600

Annual earnings(average) 46,080 69,120

The total preferred shares to be issued and the amount of goodwill to be recognized by Bats are:

a. Preferred shares: 8,640 Goodwill: P288,000

b. Preferred shares: 5,760 Goodwill: P288,000

c. Preferred shares: 2,880 Goodwill: P864,000

d. Preferred shares: 7,280 Goodwill: P864,000

ANSWER: A

II JJ TOTAL

Average annual earnings P 46,080 P 69,120 P 115,200

Divided by: Capitalized at 10%

Total stock to be issued P1,152,000

Goodwill (for Common Stock) 864,000

Preferred stock (same with Net Assets):

864,000/P100 par 8,640 shares


9. Companies A and B decide to consolidate. Asset and estimated annual earnings contributions are as
follows:

Co.A Co. B Co.C

Net asset contribution P300,000 P400,000 P700,000

Estimated annual earnings 50,000 80,000 130,000


contribution

Stockholders of the two companies agree that a single class of stock be issued, that their contributions be
measured by net assets plus allowances for goodwill, and that 10% be considered as a normal rate of return.
Earnings in excess of the normal rate of return shall be capitalized at 20% in calculating goodwill. It was also
agreed that authorizes capital stock of the new company shall be 20,000 shares with a par value of P100 a
share.

What is amount of goodwill credited to Co. A, and the total contribution of Co.B(net assets plus goodwill)”
a. P100,000; P400,000 c. P100,000; P600,000

b. P150,000;P500,000 d. P200,000; P600,000

ANSWER: C

Company A Company B

Net Asset Contribution P300,000 P400,000

Add: P50,000 P80,000

Goodwill Average/Annual Earnings

Less: Normal Earnings (10%of net 30,000 40,000


asset)

Excess earnings P20,000 P40,000

Divided by: Capitalized at 20% 20%

Goodwill P100,000 P200,000

Total contribution(stock to be P400,000 P600,000


issued

10. Malakas Company acquired all of Maganda Corporation's assets and liabilities on January 2,2013, in a
business combination. At that date, Maganda reported assets with a book value of P624,000 and liabilities of
P356,000. Malakas noted that Maganda had P40,000 of research and development costs on its books at the
acquistion date that did not appear to be of value. Malakas also determined that patents developed by
Maganda had a fair value of P120,000 but had not been recorded by Maganda. Except for building and
equipment, Malakas determined the fair value of all other assets and liabilities reported by Maganda
approximated Malakas recorded amounts. In recording the transfer of assets and liabilities to its books,
Malakas recorded goodwill of P93,000. Malakas paid P517,000 to acquire Maganda's asset and liabilities.
If the book value of Maganda's buildings and equipment was P341,000 at the date of acquisition, what was
their fair value?
a. P441,000
b. P417,000
c. P341,000
d. P417,000
Answer: B.
Solution
Computation of Fair Value
Amount paid P517,000
Book Value of assets P624,000
Book Value of liabilities. (356,000)
Book Value of net assets. P268,000
Adjustment for RandD costs. (40,000)
Adjusted book value. P228,000
Fair value of patent. 120,000
Goodwill recorded. 93,000 (441,000)
Fair value increment of
building and equipment P76,000
Book value of building and Equipment. 341,000
Fair Value of buildings and equipment P417,000
11. Richard Ltd. and Liway Ltd. are two family owned ice cream producing companies in Pampanga. Richard
Ltd. is owned by the Melad family, while the Basilio family owns Liway Ltd. The Melad family has only one son.
and he is engaged to be married to the daughter of Basilio family. Because the son currently managing Liway
Ltd., it is proposed that he be allowed to manage both companies after the wedding. As a result, it is agreed
by the two families that Richard and Ltd. should take over the net assets of Liway Ltd.

The balance sheet at Liway Ltd. immediately prior to the takeover is as follows:

Carrying Amount Fair Value

Accounts receivable P20,000 P 20,000


Inventory 140,000 125,000
Land 620,000 840,000
Buildings (net) 530,000 550,000
Farm equipment (net) 360,000 364,000
Irrigation equipment (net) 220,000 225,000
Vehicles (net) 160,000 172,000
Total assets P2,050,000

Accounts payable P80,000 P 80,000


Loan-Metrobank 480,000 480,000
Share capital 670,000
Retained earnings 820,000
Total P2,050,000
The takeover agreement specified the following details:

* Richard Ltd. is to acquire all the assets of Liway Ltd. and except one of the vehicles (having a carrying
amount of P45,000 and of fair value of P48,000) and assume all the liabilities except for the loan from
Metrobank. Liway Ltd. is then to go, into liquidation.
* Cash at P20,000, half to be paid on date of exchange and half in one year's time. The incremental
borrowing rate is 10% per annum (present value for P1 at 10% for 1 period is 0.909091).
* Supply of a patent relating to the manufacture of ice cream. This has a fair value of P60,000 but has not
been recognized in the records of Liway Ltd. because it resulted from an internally generated research
project.
* Richard Ltd. is to supply sufficient cash to enable the debt to Metrobank to be paid for and to cover the
liquidation costs of P5,500. it will also give P150. 000 to be distributed to Mr. an Mrs. Melad to assists in
paying the wedding costs.
* Richard Ltd. is also to give a piece of its own prime land to Liway Ltd. to be distributed to Mr and Mrs. Melad,
this eventually being available to be given to any offspring of the forthcoming marriage. The piece of land in
question has a carrying amount of P80,000 and a fair value of P220,000.
* Richard Ltd. is to issue 90,000 shares, these having a fair value of P14 per share, to be distributed via Liway
Ltd. to the soon to-be-married-daughter of Mr. and Mrs. Melad, who is currently a shareholder in Liway Ltd.

The takeover proceeded as per the agreement with Richard Ltd. incurring incidental acquisition costs of
P25,000, while there were P 18,000 share issue costs.

The amount of goodwill or (bargain purchase gain):

a. P45.682
b. 70,682
c. 118,682
d. P(109,818)
Answer: A
Solution
Consideration transferred:
Shares: (90.000 x P14 per share) P1,260,000
Cash: Payable Now 20,000
Deferred (P20,000 x 0.909091) 18,182
Patent 60,000
Cash (to Metrobank) 480,000
Liquidation costs 5,500
Wedding costs 150,000
Land 220,000 P2,213,682
Less: Fair value of net identifiable assets acquired.
Accounts receivable P20,000
Inventory 125,000
Land 840,000
Buildings 550,000
Farm equipment 364,000
Irrigation equipment 225,000
Vehicles ( P172,000 - P480,000) 124,000
Accounts payable (80,000) 2,168,000
Goodwill P45,682
12. The Boy George, Company acquired the net assets of the Girl Conrad Company on January 1, 2015, and
made the following entry to record the purchase:
Current Assets100,000
Equipment 150,000
Land 50,000
Buildings 300,000
Goodwill 100,000
Liabilities 80,000
Common stock,P1 par 100.000
Paid in capital in excess at par 520,000

Assuming that additional shares on January 1, 2017 would be issued on that date to compensate for any fall
in the value at Boy George common stock below P16 per share. The settlement would be to cure the
deficiency by issuing added shares based on their fair value on January 1,2017. The fair price of the shares
on January 1, 2017 was P10.

What is the additional number of shares issued on January 1, 2017 to compensate for any fall in the value at
the stock?

a. 160,000
b. 100,000
c. 60,000
d. 10,000
Answer: C
Solution
Deficiency: (P16 - P10) x100,000 shares issued to acquire P600,000
Divided by: fair value of share P 10
Additional number of shares to issued 60,000

Another example at contingencies is where the acquirer issues to the acquiree and the acquiree is
concerned that the issue of these shares may make the market price at the acquirer ’s shares decline over
time.

Therefore the acquirer may offer additional cash or shares if the market price falls below specified amount
over a specific period of time.

13. Fay acquires assets and liabilities of May Company on January 1,2016. To obtain these shares, Fay pays
P400,000 and issues 10,000 shares of P20 par value common stock on this date. Fay's stock had a fair
value of P36 per share on that date. Fay also pays P15,000 to a local investment firm for arranging the
transaction. An additional P10,000 was paid by Fay in stock issuance costs.
The book values for both Fay and May as of January 1,2016 follow. The fair value of each of Fay and May
accoubts is also included. In addition, May holds a fully amortized trademark that still retains P40,000 value.
The figures below are in thousands. Any related questions also in thousands.
May Company
Fay, Inc. Book Value Fair Value
Cash P900 P80 P80
Receivables 480 180 160
Inventory 660 260 300
Land 300 120 130
Buildings(net) 1,200 220 280
Equipment(net) 360 100 75
Accounts Payable 480 60 60
Long-term liabilities 1,140 340 300
Common Stock 1,200 80
Retained earnings 1,080 480
Assuming the combination is accounted for as an acqusition, immediately after the acquisition, in the balance
sheet of Fay:
What amount will be reported for goodwill?
a. P55 c. P70
b. 65 d. 135

Answer: A.
Consideration Transferred:
Cash P400
Shares (10,000x36) 360
Total P760
Less: Fair value of net iden. assets acquired
Cash P80
Receivables 160
Inventory 300
Land 130
Buildings(net) 280
Equipment(net) 75
Trademark 40
Accounts Payable (60)
Long-term liabilities (300) 705
Goodwill P 55

14. Using the same information in No. 1, what amount will be reported for retained earnings?
a. P1,065 c. P1,525
b. 1,080 d. 1,560

Answer: A.
Acquirer - Fay (at book value) P1,080
Less: Acquisition-related costs 15
Acquiree - May (not acquired) 0
Retained Earnings P1,065

15. Using the same information in No. 1, what amount will be reported for cash after the purchase transaction?
a. P980 c. P875
b. P900 d. P555

Answer: D.
Acquirer - Fay (at book value) P900
Less: Cash paid to acquire net assets of May 400
Acquisition-related costs 15
Stock issuance costs 10
Acquiree - May (fair value) 80
Cash P555

16. Villena Company issued its common stock for the net assets of Wynona Company in a business
combination treated as an acquisition. Villena's common stock issued was worth P 1,500,000. At the date of
combination, Villena's net assets had a book value of P 1,600,000 and a fair value of P 2,000,000 ; Wynona's
net assets had a book value of P 950,000 and a fair value of P 1,100,000. Immediately following the
combination, the net assets of the combined company should have been reported at what amount?

a. P 3,500,000 b. P 3,100,000 c. P 4,200,000 d. P 2,550,000

Answer: b. P 3,100,000

Solution:

Acquisition Cost P 1,500,000

Less: Fair Value of Identifiable

Assets Acquired 1,100,000

Goodwill P 400,000

Villena's Net Assets at Book Value 1,600,000

Wynona's Net Assets at Fair Value 1,100,000

Total Assets After Combination P 3,100,000

17. On July 1, 2014, Trence Company acquired the net assets of the Yasser Company for a price of P
42,000,000. At the acquisition date the carrying value of Yasser's net asset was P 35,000,000. At the
acquisition date a provisional fair value of the net assets was P 37,000,000. An additional valuation received
on April 30, 2015 increased the provisional value to P 38,500,000 and on July 31, 2015 this fair value was
finalized at P 40,000,000.

What amount should Trence Company present the goodwill in its statement of financial position at
December 31, 2015?

a. P 2,000,000 b. P 7,000,000 c. P 3,500,000 d. P 5,000,000

Answer: c. P 3,500,000

Solution:

Acquisition Cost P 42,000,000

Fair Value of Identifiable Assets

Acquired 38,500,000

Goodwill P 3,500,000

Items 18-19 are based on the following data:

Statement of financial position position reflecting uniform accounting procedures l, as well as faire value that
are to be used as basis of the combination are prepared on September 1, 2016 as follows:

Company AceCompany BeeCompany CidAssets


P5,250,000P6,800,000P900,000Liabilities P3,900,000P2,600,000 P480,000Capital stock, all P15 par
1,900,000 1,400,000475,000Additional paid-in capital 400,00040,000Retained earnings(deficit) (450,000)
2,400,000(95,000)Total equitiesP5,250,000P6,800,000P900,000

Ace Company shares have a market value of P22 per share. Market values is not available for shares of Bee
Company and Cid Company .

On September 1, 2016 Ace Company acquires all of the assets and assumes the liability of Bee Company and
Cid Company by issuing P200,000 shares of its stock to Bee Company andpaid 29,000 shares of its stock to
Cid Company. Ace Company pays P10,000 for registering and issuing securities and P20,000 for other
acquisition costs combination.

18. What is the goodwill to be recorded Ace Company on September 1, 2016?

a. P448,000
b. P220,000
c. P400,000
d. P418,000

19. What is the total stockholders equity in the combined statement of financial position after combination on
September 1, 2016?

a. P6,488,000
b. P3,252,000
c. P6,468,000
d. P6,458,000

Solution #18

Answer: D

Bee Company
Price paid P4,400,000
Net assets. 4,200,000
Goodwill P 200,000

Cid Company
Price paid P638,000
Net assets 420,000
Goodwill. P218,000
Total goodwill P418,000

Solution #19

Answer: A
Total Equity P1,450,000
Additional share issuance 3,435,000
Additional paid-in capital. 1,603,000
Registering and issuing fee. (10,000)
Other acquisition costs . (20,000)
Total stockholders equity P6,458,000

20. The statement of financial position of B.o.B. Company as of December 31, 2013 is as follows:
Assets Liabilities and Shareholder’s Equity
Cash 175,000 Current Liabilities 250,000
Accounts Receivable 250,000 Mortgage payable 450,000
Inventory 725,000 Ordinary Share Capital 200,000
Property, plant and equipment 950,000 Share Premium 400,000
2,100,000 Accumulated Profits 800,000
2,100,000

On December 31, 2013 the Taylor Swift Inc. bought all of the outstanding shares of B.o.B. Company for P
1,800,000 cash. On the date of acquisition, the fair market value of B.o.B.’s inventories was P 675,000, while
the fair value of B.o.B.’s property, plant equipment was P 1,100,000. The fair value of all other assets and
liabilities of B.o.B. were equal to their book values. In addition, not included above were costs in-process
research and development of B.o.B Company amounting to P 100,000.

Goodwill amounted to:

a. P 400,000 b. P 300,000 c. P 200,000 d. P -0-

Ans. C
Consideration Transferred P1,800,000
Book Value of Net Assets:
Ordinary Share Capital P200,000
Share Premium P400,000
Accumulated Profits (P800k+P100k) P900,000
Allocable excess P300,000
Increase/Decrease in assets:
Inventory (675k-725k) P50,000
P.P.E (1100k-P950K) (P150,000)
P200,000

21. Bruno Mars Company acquired Billboard Company’s net assets by issuing its own P 14 par value ordinary
shares totaling 50,000 shares at market price of P 14.55. Bruno Mars Company had the following
expenditures incurred:

Finder’s fee paid P 50,000


Pre-acquisition audit fee, 40,000
30% was paid
General administrative 15,000
costs
Doc stamp paid on 3,500
issuance for the
combination
Legal fees for the 32,000
combination paid

Audit fees for SEC 46,000


registration of share issue
SEC registration for the 10,000
share issue paid
Share issuance costs paid 10,000
(inclusive of taxes paid)

sOther indirect costs paid 16,000

The total amount debited to expense should be

a. P 153,000 b. P 156,500 c. P 195,000 d. P 191,500

Ans. D

Finder’s fee paid P 50,000


Pre-acquisition audit fee, 40,000
30% was paid
Doc stamp paid on 3,500
issuance for the
combination
Legal fees for the 32,000
combination paid
Audit fees for SEC 46,000
registration of share issue
SEC registration for the 10,000
share issue paid
Share issuance costs paid 10,000
(inclusive of taxes paid) P191,500

22. On 1 December 2015, Casio Ltd. acquired all the assets and liabilities of Aurora Ltd. With Casio Ltd.
Issuing 100, 000 shares to acquire these net assets. The fair value of Aurora Ltd.’s assets and liabilities at this
date were:

Cash P50, 000


Furniture and fittings 20, 000
Accounts receivable 5, 000
Plant 125, 000
Accounts payable 15, 000
Current tax liability 8, 000
Provision for annual leave 2, 000

The financial year for Casio Ltd. is January – December.

The fair value of each Casio Ltd. Share at acquisition date is P1.90. At acquisition date, the acquirer could
only determine a provisional fair value for the plant. On 1 March 2016, Casio Ltd. received the final value from
the independent appraisal, the fair value at acquisition date being P131, 000. Assuming the plant had further
five-year life from the acquisition date.

The amount of goodwill arising from the business combination of December 1, 2015:
a. P15, 000
b. P9, 000
c. P5, 000
d. 0

Ans: B
Solution:
Consideration transferred (100, 000 shares x P1.90) P190, 000
Less: fair value of net identifiable assets acquired:
Cash P50, 000
Furniture and fittings 2, 000
Accounts receivable 5, 000
Plant 131, 000
Accounts payable (15, 000)
Current tax liability (8, 000)
Liabilities (2, 000) 181, 000
Goodwill P9, 000

One of the problems that may arise in measuring the assets and liabilities of the acquiree is that the initial
accounting for the business combination may be incomplete by the end of the reporting period. For example,
the acquisition date may be August 18 and the end of reporting period may be August 31.

In this situation, in accordance with par. 45, the acquirer must report provisional amounts in its financial
statements. The provisional amounts will be best estimates and will need to be adjusted to fair values when
those amounts can be determined after the end of the reporting period. The measurement period in which
the adjustments can be made cannot exceed one year after the acquisition date.

The carrying amount of the plant must be calculated as if its fair value at the acquisition date has been
recognized from that date, with an adjustment to goodwill.

If the plant had a 5-year life from the acquisition dates. Casio Ltd. would have charged depreciation for 1
month in 2015. Extra depreciation of P100 being P6, 000 ÷ 5 years x 1/12 is required in 2016.

The adjusting entry at March 1, 2016 is:


(Adjustment for provisional accounting)
Plant6, 000
Goodwill6, 000

(Adjustment to depreciation due to provisional accounting)


Retained earnings, 1/1/16100
Accumulated depreciation100

If depreciation has been calculated monthly for 2016, further adjustments would be required.

23. Jane Ltd., a supplier of snooker equipment, agreed to be acquire the business of a rival firm, Mercy Ltd.
taking over all assets and liabilities as at 1 June 2016.

The price agreed upon was P40, 000, payable P20, 000 cash and the balance by the issue to the selling
company of P16, 000 fully paid shared in Jane Ltd. these shares having a fair value of P2.5 per share.

The trial balances of the two companies as at 1 June 2016 were as follows (in thousand peso):

Jane Ltd. Mercy Ltd.


Dr. Cr. Dr. Cr.
Share capital P100 P90
Retained earnings 12 P24
Accounts payable 2 20
Cash P30 -
Plant (net) 50 30
Inventory 14 26
Accounts receivable 8 20
Government bonds 12 -
Goodwill - 10
P114 P114 P110 P110

All the identifiable net assets of Mercy Ltd. were recorded by Mercy Ltd. at fair value except for the inventory
which was considered to be worth P28, 000. The plant had an expected remaining life of five years.

The business combination was completed and Mercy Ltd. went into liquidation. Cost of liquidation amounted
to P1, 000. Jane Ltd. incurred incidental costs of P500. Cost of issuing shares in Jane Ltd. were P400.

The amount of goodwill:

a. P0
b. P2, 000
c. P2, 900
d. P3, 900

Ans. :B
Solution:

Consideration transferred:
Cash P20, 000
Shares:16, 000 shares x P2.50 40, 000 P60, 000

Less: fair value of net identifiable assets acquired:


Plant 30, 000
Inventory 28, 000
Accounts receivable 5, 000
Plant 20, 000
Accounts payable (20, 000) 58, 000
Goodwill P2, 000

It should be noted that acquisition-related costs is not the same with liquidation-related costs even though the
consequence of acquisition is liquidation of the acquiree. Any costs of liquidation or of similar item paid or
supplied by the acquirer should be part of the consideration transferred for reason that it was intended to
complete the process of liquidation. The reason for such inclusion is that the consideration received from the
acquirer may be used to pay for liabilities not assumed by the acquirer and for liquidation expenses which is
tantamount for unrecorded liabilities from liquidation point of view. These items should not be confused with
acquisition-related costs as noted earlier which are considered outright expenses. Further, any liquidation
costs or similar item which was not of the same situation as mentioned above should be treated as expenses.

When it liquidates, costs of liquidation paid by the acquiree should be for the account of the acquire and will
be eventually transferred to stockholders’ equity account. This payment made should considered expenses
by the acquiree in the process of liquidation not unlike payment supplied and made by the acquirer which is
intended for any unrecorded expenses.

Faith Company is acquiring the net assets of Love Company for an agreed upon price of P1000,000 on
April 1,2014. The value was tentatively assigned as follows:

Current Assets P 100,000


Land 70,000
Equipment - 5 year life 300,000
Building -20 year life 500,000
Current Liabilities (200,000)
Goodwill 230,000

Values were subject to change during the measurement period. Depreciation is taken to the nearest
month. The measurement period expired on April 1, 2015 at which time the fair value of the
equipment and building as of acquisition date were revised to 280,000 and 600,000, respectively.
24. How much total depreciation expense will be recorded for 2015.
a. 85,000
b. 86,000
c. 83,500
d. 86,500

Ans. B
Equipment 280,000/5 56,000
Building 600,000/20 30,000
86,000
25. How much goodwill is presented in 2015 statement of financial position?
a. 230,000
b. 180,000
c. 150,000
d. 200,000

Ans. C
Agreed price 1,000,000
Less: fair value of net assets
(1,050,000-200,000) 850,000
150,000

26. Westport Ltd. a suplier of snooker equipment, agreed to acquire the business of a rival firm, Manukau Ltd.
taking over all assets and liabilities as at 1June 20x4.
The price agreed upon was P40,000, payable P20,000 cash and the balance by the issue to the selling
company of P16,000 fully paid shares in Westport Ltd. these shares having a fair value of P2.50 per share.
The trial balances of the two companies aa at 1 June 20x4 were as follows:
Westport Ltd Manukau Ltd.

Share capital P100,000 P 90,000


Retained earning 12,000 P 24,000
Accounts payables 2,000 20,000
Cash P 30,000 -
Plant 50,000 30,000
Inventory 14,000 26,000
Accounts receivable 8,000 20,000
Government bonds 12,000 -
Goodwill ---- 10,000
P 114,000 P 114,000 P 110,000 P 110,000

All the identifiable net assets of Manukau Ltd. were recorded by manukau Ltd. At fair value except for the
inventory which was considered to be worth P28,000. The plant had an expected remaining life of five years.
The business combination was completed and Manukau Ltd. went into liquidation. Westport Ltd. Incurred
incidental costs of P500 in relation to the acquisition cost. Cost of issuing shares in Wesport Ltd. were P400.
The amount of goodwill to:

A. Nil or zero
B. P2,509
C. P2,900
D. P3,900
ANSWER: B

Cost of investment {20,000 + (16,000


P 60,500
shares x P2.50) + 500 incidental cost}
Less: markt value of net assets acquired: P 30,000
Plant 28,000
Inventory 5,000
Account receivable 20,000
Plant (20,000)
Accounts payable
58,000
Goodwill P 2,500

27. Bats Inc., a new corporation formed and organized because of the recent consolidationof II Inc. and JJ
Inc., shall issue 10% participating preferred stocks with a par value of P100 for all II andJJ net assets
contributions, and common shares with a par value of P50 for the difference between the total shares to be
issued and the preffered shared to be issued. The total shares to be issued by Bats shall be equivalent to
average annual earnings capitalized at 10%. Relevant data on II and JJ follows:
II JJ
Total assets.................................... P720,000 P921,600
Total liabilities................................ 432,000 345,600
Annual earnings (average)............ 46,080 69,120
The total preferred shares to be issued and the amount of goodwill to be recognized by Bats are:

A. Preferred shares: 8,640 Goodwill: P288,000


B. Preferred shares: 5,760 Goodwill: P288,000
C. Preferred shares: 2,880 Goodwill: P864,000
D. Preferred shares: 7,280 Goodwill: P864,000

ANSWER: A

II JJ Total
Average annual arnings P 46,080 P 69,120 P 115,200
Divided by: capitalized at 10%
Total stock to be issued P 1,152,000
Less: net assets (for P/S) 864,000
Goodwill (for common
P 288,000
stock)
Preferred stock (same with

Net assets):864,000/100 8,640 shares

28. Cormorant Corporatlon paid 800,000 for a 40% Interest in Plumage Company on January 1, 2005 when
Plumage's stockholder's equity was as follows:

10% cumulative preferred stock, $100 par S 500,000


Common stock, $10 par value S 300,000
Other paid-In capital S 400,000
Retained earnings S 800,000
Total stockholders’ equity S 2,000,000

On this date, the book values of Plumage's assets and liabilities equaled their fair values and there were no
dividends In arrears. Goodwill from the investment is
a.S 0.
b. 150,000.
c. 200,000.
d. None of the above ls correct.

Answer: d

Cost of Cormorant's investment: $ 800,000


Less: book value acquired:
Total equity $ 2,000,000
Less: Preferred equity $ 500,000
Net common equity $ 1,500,000
x percent acquired 40%
= Plumage book value $ 600,000 $ 600,000
Goodwill $ 200,000
29. On February 5, Pryor Corporation paid $1,600,000 for all the issued and outstanding common stock of
Shaw, Inc., in a transaction properly accounted for as an acquisition. The book values and fair values of
Shaw's assets and liabilities on February 5 were as follows:

Book Value Fair Value


Cash $ 160,000 $160,000
Receivables (net) 180,000 180,000
Inventory 315,000 300,000
Plant and equipment (net) 820,000 920,000

Liabilities (350,000) (350,000)


Net assets $1,125,000 $1,210,000

What is the amount of goodwill resulting from the business combination?

a) $-0-.
b) $475,000.
c) $85,000.
d) $390,000.

Answer: d

FV of consideration transferred $ 1,600,000


Less: FV of Net Assets $ 1,210,000
Goodwill $ 390,000

On January 1, 20x5, the fair values of Crème’s net assets were as follows:

Current Asset P100,000


Equipment 150,000
Land 50,000
Buildings 300,000
Liabilities 80,000
30. On January 1, 20x5, Brulee Company purchased the net assets of the Crème Company by issuing
100,000 shares of its P1 par value stock when the fair value of the stock was P6.20. It was further agreed that
Brulee would pay an additional amount on January 1, 20x7, if the average income during the 2-year period of
20x5-20x6 exceeded P80,000 per year. The expected value of this consideration was calculated as P184,000;
the measurement period is one year. What amount will be recorded as goodwill on January 1, 20x5?
a. Zero c. P180,000
b. P100,000 d. P284,000

Ans: d

Consideration transferred
Shares: (100,000 shares x P6.20) P620,000 Contingent
consideration 184,000
Total P804,000
Less: Current Assets (at fair values) P100,000
Equipment 150,000
Land 50,000
Buildings 300,000
Liabilities ( 80,000) 520,000
Goodwill P284,000

31. On July 1, 20x5 The Straw Company acquired 100% of the Berry Company for a consideration transferred
of P160 Million. At the acquisition date the carrying amount of Berry’s net assets was P100 Million. At the
acquisition date a provisional fair value of P120 Million was attributed to the net assets. An additional valuation
received on May 31, 20x6 increased this provisional fair to P135 Million and on July 30, 20x6 this fair value
was finalized at P140 Million. What amount should Straw present for goodwill in its statement of financial
position on December 31, 20x6, according to PFRS 3 Business Combinations?
a. P20 million c. P50 million
b. P25 million d. P60 million

Ans: b

Consideration transferred P160 million


Fair Value on May 31, 20x6 135 million
Goodwill P 25 million

32. Hazel Corp. was merged into Sebastian Corp. in a combination properly accounted for as acquisition of
interest. Their condensed sheets before the combination show:

Sebastian Hazel

Current assets…………………………………………….. P1,144,000 P 813,800

Plant and equipment, 2,327,000 520,000


net………………………………...
Patents…………………………………………………….. - 130,000
Total assets……………………………………………….. P3,471,000 P1,463,800

Liabilities…………………………………………………. P1,352,000 P 85,800


..
Capital stock, par 1,300,000 650,000
P100…………………………………..
Additional paid-in 195,000 195,000
capital………………………………..
Retained 624,000 533,000
earnings……………………………………......

Total Liabilities and Equity………………………………. P3,471,000 P1,463,800

Per independent appraiser’s report, Hazel’s assets have fair market values of P826, 800 for current assets,
P624, 000 for plant and equipment and P169, 000 for patents. Hazel’s liabilities are properly valued.
Sebastian purchases Hazel’s net assets for P1, 534,000. How should the difference between the book value
of Hazel’s net assets and the consideration paid by Sebastian be considered?

a. Goodwill: P 0 ; Increase in Assets: P156,000

b. Goodwill: P 0 ; Increase in Assets: P312,000

c. Goodwill: P169,000 ; Increase in Assets: P156,000

d. Goodwill: P169,000 ; Increase in Assets: P 78,000

ANSWER: (a)

Consideration transferred………………... P1,534,000

Less: Market value of net assets acquired, excluding GW:

Current P826,800
assets……………………...
Plant and 624,000
equipment……………….
Patents……………………………… 169,000

Liabilities……………………………. (85,800) 1,534,000

Goodwill…………………………………… P -0- (a)


Book Fair Increase

Value Value (Decrease)

Current Assets………………. P813,800 P826,800 P 13,000

Plant and Equipment……….. 520,000 624,000 104,000

Patents……………………….. 130,000 169,000 39,000

Increase in assets…………… P156,000 (a)

33. On December 2015, Agulan Co. acquired all the assets and liabilities of Toquero Co. with Agulan
Co. issuing 150,000 shares to acquire these net assets. The fair value of Toquero Co.’s assets and
liabilities at this date were:

Cash…………………………………………………………… P75,000

Accounts receivable…………………………………………. 7,500

Fix and Furnitures……………………………………………. 30,000

Plant and Equipment………………………………………… 187,500

Accounts payable…………………………………………….. 22,500

Current tax liability……………………………………………. 12,000

Provision for annual leave…………………………………… 3,000

The financial year for Agulan Co. is January – December.

The fair value of each Agulan Co. share at acquisition date is P2. At acquisition date, the acquirer
could only determine a provisional fair value for the plant and equipment. On March 1, 2016, Agulan
Co. received the final value from the independent appraisal, the fair value at acquisition date being
P196,500. Assuming the plant and equipment had a further five-year life from the acquisition date.

The amount of goodwill arising from the business combination at December 1, 2015:

a. P 0
b. P18,750

c. P37,500

d. P30,500

ANSWER: (c)

Consideration transferred (150,000 shares x P2) P300,000

Less: Fair value of net identifiable assets


acquired:

Cash……………………………………………. P 75,000

Accounts receivable………………………….. 7,500

Fix and Furniture……………………………… 30,000

Plant and Equipment………………………… 187,500

Accounts payable…………………………….. ( 22,500)

Current tax liability…………………………… ( 12,000)

Liabilities………………………………………. ( 3,000) 262,500

Goodwill……………………………………………… P37,500

34. Homer Ltd. is seeking to expand its share of the widgets market and has negotiated to take over the
operations of Tan Ltd. on January 1, 20x4. The balance sheets of the two companies as at December 31,
20x4 were as follows:
Homer Tan
Cash P 23,000 P 12,000
Receivables 25,000 34,700
Inventory 35,500 27,600
Freehold Land 150,000 100,000
Buildings (net) 60,000 30,000
Plant and equipment (net) 65,000 46,000
Goodwill 25,000 2,000
P383,500 P252,300

Accounts payable P 56,000 P 43,500


Mortgage loan 50,000 40,000
Debentures 100,000 50,000
Common stock, 100,000 shares 100, 000
Common stock, 60,000 shares 60,000
Additional paid-in capital 28,500 26,800
Retained earnings 49,000 32,000
P 383,500 P 252,300

Homer Ltd. is to acquire all the assets, except cash of Tan Ltd. The assets of Tan are all recorded at fair value
except:
Fair Value
Inventory P 39,000
Freehold land 130,000
Buildings 40,000

ln exchange, Homer Ltd. is to provide sufficient extra cash to allow Tan Ltd. to repay all of its outstanding
debts and its liquidation costs of P2,400, plus two fully paid shares in Homer Ltd. for every three shares held
in Tan Ltd. The fair value of a share in Hastings Ltd. is P320. An investigation by the liquidator of Tan Ltd.
reveals that on December 31, 20x3, the followmg outstanding debts were outstanding but had not been
recorded:

Accounts payable P1,600


Mortgage interest 4,000

The debentures issued by Tan Ltd. are to be redeemed at a 5% premium. Costs of issuing the shares were
P1,200.
The excess of fair value of net assets over cost or gain on acquisition that will be recognized immediately in
the income statement is:

a. Nil or Zero
b. P17,700
c. P29,700
d. P34,300

ANSWER: C

Consideration transferred:
Shares: 2/3 x 60,000 x P3.20. 128,000
Cash
Accounts payable. 45,100
Mortgage and interest 44,000
Debentures and premium 52,500
Liquidation expenses 2,400
144,000
Cash held (12,000) 132,000
260,000
Less: Fair value of assets and liabilities acquired:
Accounts receivable P34,700
Inventory 39,000
Freehold land 130,000
Buildings 40,000
Plant and equipment 46,000 289,700
Bargain Purchase Gain P 29,700
35.. Westport Ltd., a supplier of snooker equipment, agreed to acquire the business of a rival firm, Manukau
Ltd. taking over all assets and liabilities as at 1 June 20x4.

The price agreed upon was P40,000, payable P20,000 cash and the balance by the issue to the selling
company of 16,000 fully paid shares in Westport Ltd. these shares having a fair value of P2.50 per share.

The trial balances of the two companies as at 1 June 20x4 were as follows.

Westport Ltd. Manukau Ltd.


Dr. Cr. Dr. Cr.
Share capital P100,000 P 90,000
Retained earnings 12,000 P 24,000
Accounts payable 2,000 20,000
Cash P30,000 -
Plant (net) 50,000 30,000
Inventory 14,000 26,000
Accounts receivable 8,000 20,000
Government bonds 12,000 -
Goodwill - - .
P114,000 P114,000 P110,000 P110,000
All the identifiable net assets of Manukau Ltd. were recorded by Manukau Ltd. at fair value except for the
inventory which was considered to be worth P28,000. The plant had an expected remaining life of five years.

The business combination was completed and Manukau Ltd. went into liquidation. Westport Ltd. incurred
incidental costs of P500 in relation to the acquisition costs. Costs of issuing shares in Westport Ltd. were
P400. The amount of goodwill to:

a. Nil or zero
b. P2,500
c. P2,900
d. P3,900

ANSWER: B

Cost of Investment
[P20,000 + (16,000 shares x P2.50) + P500, incidental costs) P 60,500
Less: Market value of net assets acquired:
Plant P 30,000
Inventory 28,000
Accounts receivable 5,000
Plant 20,000
Accounts payable ( 20,000) 58,000
Goodwill P 2,500

When it liquidates, costs of liquidation paid by the acquiree should be for the liquidation account of the acquiree
and will eventually be transferred to shareholders’ equity account. Any costs of liquidation paid or
supplied by the acquirer should be capitalized as cost of acquisition which stent with the cost model under
PFRS No. 3 in measuring the cost of the combination.
Any direct costs of acquisition should be capitalizable under the cost model reiterated in PFRS No. 3 Phase I.
This model in PFRS No. 3 will be amended under Phase II (pending implementation possibly until early 2008),
wherein all direct costs will be outright expense. Costs of issuing shares will be debited to share premium or
APIC account.

Any costs of liquidation paid or supplied by the acquirer should be capitalized as cost of acquisition which is
consistent with the cost model under PFRS No. 3 in measuring the cost of the combination. The fair values of
liabilities undertaken are best measured by the present values of future cash outflows. Intangible assets are
recognized when its fair value can be measured reliably. Assets other than intangible assets must be
recognized if it is probable that the future economic benefits will flow to the acquirer and its fair value can be
measured reliably.

36. Mango Company acquired Apple Company on January 2, 2016 by issuing common shares. All of Apple’s
assets and liabilities were immediately transferred to Mango Company which reported total par value of
shares outstanding of P218,400 and P327,600 and additional paid-in capital of P370,000 and P650,800
immediately before and after the business combination, respectively.
Assuming that Mango’s common stock had a market of P25 per share at the time of acquisition, what
number of shares was issued?

a. 15,600
b. 10,000
c. 15,600
d. 10,000

Answer: C
Par value of shares outstanding following merger P327,600
Paid-in capital following merger 650,800
Total fair value of paid-in capital P978,400
Par value of shares outstanding before merger P218,400
Paid-in capital before merger 370,000 (588,400)
Increase in par value and paid-in capital P390,000
Divided by price per share P25
Number of shares issued 15,600

37. The stockholder’s equities of Milkita Corporation and Keanu Company at June 1,2016 before
combination were as follows:
Milkita Keanu

Capital Stock, P100 par value P10,000,000 P3,000,000

APIC 50,000 -

Retained Earnings 5,000,000 1,000,000

37. On June 2,2016, Milkita Corporation issued 50,000 of its unissued shares with a market value of P103 per
share for the assets and liabilities of Keanu Company. On the same day Milkita Corporation paid P100,000 for
legal fees, documentary stamp tax of P20,000 and P190,000 for SEC registration fees of equity securities.

Shareholder’s equity would include :


a. P15,000,000 Capital Stock ; P4,900,000 Retained earnings ; P10,000 Stock issuance cost
b. P15,000,000 Capital stock ; P10,000 APIC ; P4,880,000 Retained earnings
c. P15,150,000 Capital Stock ; P50,000 APIC ;P 4,690,000 Retained earnings
d. P15,000,000 Capital Stock ; P200,000 APIC ; P4,690,000 Retained earnings
Answer : A
Capital stock:
Before combination P10,000,000
Issued at par (50,000 x P100) 5,000,000 P15,000,000
APIC:
Before combination 50,000
Issuance (P3 x 50,000) 150,000
Documentary stamp tax ( 20,000 )
SEC Registration fees ( 180,000) --0—
Retained earnings:
Before combination 5,000,000
Legal fees ( 100,000 ) 4,900,000
Stock issuance cost (P190,000+20,000-200,000) ( 10,000 )
Stockholder’s equity P19,890,000

38. Red Company issued its common stock for the net assets of Blue Company in a business
combination treated as acquisition. Red’s common stock issued was worth P1,500,000. At the date of
combination, Red’s net assets had a book value of P1,600,000 and a fair value of P1,800,000. Blue’s
net assets had a book value of P700,000 and a fair value of P850,000. Immediately following the
combination, the net assets of the combined company should have been reported at what amount?
a. P3,000,000
b. P2,400,000
c. P3,100,000
d. P1,850,000

ANSWER: C
Rationale
Acquisition Cost P1,500,000
Net assets acquired 850,000
Goodwill 650,000
Red’s net assets @BV 1,600,000
Blue’s net assets @FV 850,000
Total net assets P3,100,000

39. Mata Inc. purchased all of the net assets of Torralba Company on February 1,2015 by issuing
8,000 shares of its P20 par common stock. At the time, the stock was selling for P40 per share.
Direct costs associated with consummating the combination totalled P5,000. Under IFRS 3, what total
amount should the net assets acquired be recorded by Mata Inc. Assuming the contingent
consideration of P7,000 is determined?

ANSWER: C
Rationale (8,000 shares X 40 = P320,000 + 7,000 contingent consideration = P327,000)
40. Payla Co. Will issue share of P12par common stock for the net assets of Talisay Co. Payla’s common
stock has a current market value of P40 per share. Talisay balance sheet accounts follow:
Current Assets P500 000 Common stock, parP4 (P80 000)
Property and equipment 1 500 000 Additional paid-in-capital (320 000)
Liabilities (400 000) Retained earnings (400 000)
Talisay current assets and property and equipment, respectively, are appraised of P 400 000 and P1600 000;
it’s liabilities are fairly valued. Accordingly, Payla Co. Issued shares of it’s common stock with total market
value equal to that of Max net assets. To recognize goodwill of P200 000, how many shares were issued?
a. 55 000 c. 40 000
b. 45 000 d. 50 000
Solution:
ANS: B
Fair value of net identifiable assets acquired:
Current assets P 500 000
Property and equipment 1 500 000
Liabilities (400 000)
FMV of net assets P1 600 000
Add: Goodwill 200 000
Consideration transferred P1 800 000
Divided By: Current market value per share P 40 Number
of shares issued 45 000

41. Companies of P and J decide to consolidate. Asset and estimated annual earnings contributions are as
follows:
Co. P Co. J Total
Net asset contribution P400 000 P350 000 P750 000
Estimated annual earnings contribution 80 000 70 000 150 000

Stockholders of the two companies agree that a single class of stock be issued, that their contributions be
measured by net assets plus allowances for goodwill, and that 10% be considered as a normal rate of return.
Earnings in excess of the normal rate of return shall be capitalized at 20% in calculating goodwill. It was also
agreed that the authorized capital stock of the new corporation shall be 20,000 shares with a par value of
P100 a share.
(1)The total contribution of Co. J(net assets plus goodwill), and (2)The amount of goodwill credited to Co. A:
a.(1)P475 000;(2)P100 000 c.(1)P525 000;(2)P200 000
b.(1)P500 000;(2)P150 000 d.(1)P600 000;(2)P100 000
Solution:
ANS: C
Company A Company B
Net Asset Contributions P400 000 P350 000
Add: Goodwill
Average/Annual Earnings P 80 000 P 70 000
Less: Normal Earnings
(10% on Net Asset) 40 000 35 000
Excess Earnings P 40 000 P 35 000
Divided by: Capitalized at 20% 20%
Goodwill P 200 000(c) P 175 000
Total Contribution (stock to be issued) P 400 000 P 600 000(c)

42. AB Corporation was merged into CD Corporation in a combination properly accounted for as acquisition
of interests. Their balance sheets before the combination are as follows:
AB Corp.
Current Assets .................................................................. P 8,352,950
Plant and Equipment,net ................................................... 6,450,700
Patents............................................................................ -

Total Assets........................................................................ P 14,803,650

Liabilities.......................................................................... P 5,713,650
Capital Stock,par P100 .................................................... 4,600,000
Additional paid-in capital .................................................. 950,000
Retained Earnings ............................................................. 3,540,000

Total Liabilities and Equity ........................................ P14,803,650

CD Corp.
Current Assets .................................................................. P 7,505,000
Plant and Equipment,net .................................................. 3,130,450
Patents................................................................................ 153,800

Total Assets...................................................................... P10,789,250

Liabilities.......................................................................P 939,000
Capital stock,par P100 .......................................................3,400,000
Additional paid-in capital .................................................... 950,000
Retained Earnings .............................................................. 5,500,250

Total Liabilities and Equity ......................................... PP10,789,250

Per-independent appraiser’s report, the fair market value of CD’s current assets is P7,808,000; plant and
eqipment is P3,452,000; and patents P286,900. Liabilities of CD Corporation are properly valued. AB
Corporation purchases the net assets of CD Corporation for P10,607,900. How should the difference
between the book value of CD Corporation’s net assets and the consideration paid by AB Corporation be
considered?

A. Goodwill: P 286,900; Increase in Assets: P 757650

B. Goodwill: P 286,900; Increase in Assets: P 303,000

C. Goodwill: P 0; Increase in Assets: P 303,000

D. Goodwill: P 0; Increase in Assets: P 757,650

Answer: D

Consideration Transferred ................................................................................................... P10,607,900


Less: Market value of net assets acquired, excluding GW:
Current Assets ........................................................... P7,808,000
Plant and Equipment ................................................. 3,452,000
Patents .......................................................................... 286,900
Liabilities................................................................. ( 939,000) 10,607,900
Goodwill............................................................................ P -0- (D)

Current Assets Plant and Equipment Patents


Book value P 7,505,000 P 3,130,450 P 153,800
Fair Value 7,808,000 3,452,000 286,900

Increase(Decrease) P 303,000 P 321,550 P 133,100


in assets

Current Assets P 303,000


Plant and Equipment 321,550
Patents 133,100

Increase in Assets P 757,650 (D)

43. Companies XX, YY, and ZZ decide to consolidate. The parties to a consolidation have the following data:

Net Assets Average annual earnings


XX Co...................... P 6,800,000 P 680,000
YY Co. .................... 3,000,000 400,000
ZZ Co. .................... 10,200,000 920,000

The parties collectively agreed that the new corporation, RR Co. Will issue a single class of stock based on
the earnings ratio. What is the stock distribution ratio to companies XX, YY,and ZZ respectively?

A. 34:15:51
B. 33:15:52
C. 34:20:46
D. 33:21:46

Answer: C

XX: P 680,000 680,000/2,000,000 = 34%


YY: 400,000 400,000/2,000,000 = 20%
ZZ: 920,000 920,000/2,000,000 = 46%

P 2,000,000 100%

44. Pak company’s owns 50% of Ganern Company’s cumulative preference shares and 30% of its ordinary
shares.Ganern’s shares outstanding at December 31, 2016 include of 10% cumulative preference shares and
P40,000,000 of ordinary shares.
Ganern reported profir of P8,000,000 for the year ended December 31,2016. Ganern declared and paid
P1,500,000 preference shares during 2016. Ganern paid no preference shares dividend during 2015. On
January 31,2017, prior to the date that the financial statements are authorized to issue, Ganern distributed
10% ordinary share dividend.
How much is the total amount to be recognized by Pak Company in its 2016 profit and loss related to these
investment?
a. P2,450,000
b. P2,600,000
c. P2,700,000
d. P2,850,000
Answer: D
Solution:
Ganern profit P8,000,000
Multiplied by: pak company’s interest 30%
Pak Company share in Ganern’s profit P2,400,000
Dividends declared and paid 1,500,000
Multiplied by: pak company’s interest 30%
Dividend income 450,000
P2,850,000
45. Companies T, G, B, parties to consolidation have the following data:

T Co. G. Co. B. Co
Net Assets………………….. P400, 000 P600, 000 P1, 000, 000
Average annual earnings…. 60, 000 60, 000 80, 000

The parties collectively agreed that the new corporation, RC Co. will issue a single class of stocks based on
the earnings ratio. What is the stock distribution ratio to companies T, G, B, respectively?

a. 20:30:50 c. 30:40:30
b. 30:30:40 d. 40:40:30

ANSWER:
Fraction
T: P60, 000 6/20 = 30%
G: 60, 000 6/20 =30%
B: 80, 000 8/20 =40%
P200, 000 100%
46. When should a business combination be undertaken?
A. When a positive net present value is generated to the shareholders of an acquiring firm.
B. When the two firms are in the same line of business, but economies of scale cannot be attained by
the acquiror.
C. When two firms are in different lines of business, creating diversification.
D. When cash will be paid for the acquired firm's stock.

Answer: A.

A business combination is beneficial when the result is a positive NPV. This effect results from synergy, which
exists when the value of the combined firm exceeds the sum of the values of the separate firms. It can be
determined by using the risk-adjusted rate to discount the change in cash flows of the newly formed entity. If
a positive net present value is generated, a combination is indicated.

Answer (B) is incorrect because a combination is indicated if economies of scale can be attained. Answer
(C) is incorrect because diversification may or may not result in a positive NPV. Answer (D) is incorrect
because some beneficial combinations involve exchanges of stock.

47. Which of the following statements is most correct?


a. A firm acquiring another firm in a horizontal merger will not have its required rate of return affected
because the two firms will have similar betas
b. In most mergers, the benefits of synergy and the price premium the acquirer pays over market price are
summed and then divided equally between the shareholders of the acquiring and target firms
c. Financial theory says that the choice of how to pay for a merger is really irrelevant because, although it
may affect the firm’s capital structure, it will not affect the firm’s overall required rate of return
d. The primary rationale for any operating merger is synergy, but it is also possible that mergers can include
aspects of both operating and financial mergers
Answer : D
48.A gain should be reported on an acquisition if:

a. The fair value of the consideration paid is less than the book value of the net assets acquired.
b. The fair value of the consideration paid plus the present value of any earnings contingency is
less than the book value of the net assets acquired.
c. The fair value of the consideration paid is less than the fair value of net assets acquired plus
the fair value of identifiable intangibles acquired.
d. The fair value of the consideration paid plus the present value of any earnings contingency is
less than the fair value of identifiable net assets acquired.

ANS: D
49. The following statements pertaining to business combination are not true except:

a. The pooling of interest method recorded the assets and liabilities of the acquired company at their fair
values.

b. Statutory merger refers to the combining of two or more existing legal entities into one new legal entity
wherein the previous companies are dissolved and are then replaced by the new continuing company.

c. In a stock acquisition, the parent and the subsidiary has their own separate financial records and
statements for external financial reporting purposes.

d. The acquiring enterprise may inherit the acquired firm's inefficiencies and problems together with its
inadequate resources.

Answer: d

50. The cost of registering equity securities in a business combination should be recorded as;
a. An income of the period
b. an expense of the period
c. Deduction from additional paid in capital
d. Part of the cost of the stock acquired

Answer: C

DATE OF ACQUISITION
1. Jericel Company had common stock of P350,000 and retained earnings of P490,000. Cathrene Inc.
had common stock of P700,000 and retained earnings of P980,000. On January 1, 2016, Cathrene
issued 24,000 shares of common stock with a P12 par value and a P35 fair value for all of Jericel
company’s common stock. This combination was accounted for as an acquisition. Immediately after
the combination, what was the consolidated net asset?
a. P280,000
b. P2,520,000
c. P1,680,000
d. P1,190,000
ANS: A

Consolidated Stockholder’s Equity


Acquirer (Parent-Cathrene), book value
(P700,000 + P980,000) P1,680,000
Add: Newly issued shares
(34,000 x P35 fair value) 1,190,000
Acquiree (Subsidiary-Jericel) eliminated
in preparing consolidated balance sheet. 0
P2,870,000
2 .On January 1, 2016, Park Corporation and Strand Corporation and their condensed balance sheet are as
follows:

Park Corp. Strand Corp.

Current Assets…………………………………. 70,000 20,000


Non-current Assets……………………………. 90,000 40,000

Total Assets…………………………………… 160,000 60,000


Current Liabilities……………………………..30,000 10,000
Long term debt……………………………….. 50,000 -
Stockholders’ Equity…………………………. 80,000 50,000

Total Liabilities and Equities 160,000 60,000

On January 2, 2016.Park Corporation borrowed 60,000 and used the proceeds to obtain 80% of the
outstanding common shares of Strand Corporation. The acquisition price was considered proportionate to
Strand’s fair value. The 60,000 debt is payable in 10 equal annual principal payments, plus interest, beginning
December 31, 2016. The excess fair value of the investment over the underlying book value of the acquired
net assets is allocated to inventory (60%) and to goodwill (40%).

On a consolidated balance sheet as of January 2, 2016, what should be the amount for each of the following?

The amount of goodwill using proportionate basis (partial):

A. Using the same information in No.60, the amount of goodwill using full fair value.(full/gross-up) basis:
a. P 0
b. 8,000
c. 10,000
d. 20,000

ANS:C
Fair value of Subsidiary:
Fair value of consideration given: 60,000 x 80% 75,000
Less :Book value of Net Assets/ Stockholders’
Equity of Subsidiary 50,000
Allocated Excess 25,000

Less: Over/ Undervaluation of Assets and Liabilities:


Increase in Inventory (25,000 x 60%= 15,000 x 100%) 15,000

Goodwill (full/gross-up) 10,000

*100% increase of inventory should amount to 15,000/80%

B .Using the same information in No.60, the amount of stockholders’ equity using full fair value (full/gross up
goodwill) proportionate basis to determine non-controlling interest should be:
a. 80,000
b. 93,000
c. 95,000
d. 130,000

ANS:C
Park stockholders equity 80,000
Non-controlling interest (full goodwill)
Strand stockholders’ equity 50,000
Add: Adjustments to reflect fair value -
inventory 15,000

Strand stockholders’ equity at FV 65,000


Non-Controlling interests 20% 13,000

Non-Controlling interests (partial) 93,000


Add: Non-Controlling interest in full goodwill
(10,000-8,000) 2,000

Consolidated Stockholders’ Equity 95,000

3. On January 2, 2011, Pare Co. purchased 75% of Kidd Co’s outstanding common stock. On that date, the
fair value of the 25% noncontrolling interest was P35,000. During 2011, Kidd had net income of P20,000.
Selected balance sheet data at December 31,2011, is as follows:

Pare Kidd

Total assets P420,000 P180,000

Liabilities P120,000 P60,000

Common stock 100,000 50,000

Retained Earnings 200,000 70,000

During 2011 Pare and Kidd paid cash dividends of P25,000 and P5,000 respectively, to their shareholders.
There were no other intercompany transactions.

In Pare’s December 31,2011 consolidated balance sheet, what amount should be reported as noncontrolling
interest in net assets?

a. P30,000

b. P35,000

c. P38,750

d. P40,000

ANSWER: C

Fair value of noncontrolling interest P35,000


Plus: Share of net income (25% x 20,000) 5,000
Less: Share of dividends (25% x 5,000) (1,250)
Noncontrolling interest P38,750
4. When it purchased Sutton, Inc. on January 1, 20x1 Pavin Corporation issued 500,000 shares of its P5 par
voting common stock. On that date the fair value of those shares totaled P4,200,000. Related to the
acquisition, Pavin had payments to the attorneys and accountants of P200,000, and stock issuance fees of
P100,000. Immediately prior to the purchase, the equity sections of the two firms appeared as follows:

Pavin Sutton

Common Stock P4,000,000 P700,000

Paid in capital in excess of par 7,500,000 900,000

Retained earnings 5,500,000 500,000

Total P17,000,000 P2,100,000

Immediately after the purchase, the consolidated balance sheet should report paid in capital in excess of par
of.

a. P8,900,000

b. P9,100,000

c. P9,200,000

d. P9,300,000

ANSWER: B

FV, stocks issued P 4,200,000


Less: Par value of stocks issued (500,000 shares x 2,500,000
P5)

APIC P 1,700,000
Add: APIC of P 7,500,000
Less: Stock issuance cost 100,000
P 9,100,000

5 .The Moon Company acquired a 70% interest In The Swan Company for P1,420,000 when the fair value
of Swan's identifiable assets and labilities was P1,200,000. Moon acquired a 65% interest In The Homer
Company for P300,000 when the fair value of Homer's identifiable assets and liabilities was P640,000.
Moon measures non-controlling interest at the relevant share of the identifiable net assets at the
acquisition date. Neither Swan nor Homer had any contingent liabilities at the acquisition date and the
above fair values were the same as the carrying amounts in their financial statements. Annual impairment
reviews have not resulted In any impairment losses being recognized.
Under PFRS 3 Bussiness combinations, what figures in respect of goodwill and of gains on bargain
purchases should be included in Moon's consolidated statement of financial position?
a. Goodwill: P580,000: Gains on the bargain purchases: P116,000
b. Goodwill: Nil or zero: Gains on the bargain purchases: P116,000
c. Goodwill: Nil or zero; Gains on the bargain purchases: Nil or zero
d. Goodwill: P580,000: Gains on the bargain purchases: Nil or zero
Answer: D
Solution
Fair value of subsidiary - Swan
Consideration transferred P1,420.000
less: Fair value at identifiable assets and liabilities of Swan
(70% x P1.2 million) 840.000
Goodwill (partial) P580,000
"Goodwill is carried as on asset in the consolidated statement of financial position."

Fair value of Subsidiary Homer

Consideration transferred P 300. 000

less: Fair value at identifiable assets and Liabilities of Homer


(65% x P640 000) 416,000

Gain on bargain purchases P(116,000)

6. Questions A and B are based on the following:


Winston has the following account balances as of February 1, 2014:
Inventory P 600,000 Common stock (P10 par value) P 800,000
Land 500,000 Retained earnings, Jan. 1,2014 1,100,000
Buildings (net) (FV P1,000,000) 900,000 Revenues 600,000
Expenses 500,000

Arlington pays P1.4 million cash and issues 10,000 shares of is P30 par value common stock (valued at
P80 per share) for all of Winston’s outstanding stock and Winston is dissolved. Stock issuance costs
amount to P30,000. Prior to recording these newly issued shares, Arlington reports a Common Stock
account of P900,000 and Additional Paid-in Capital of P500,000.

A. Determine the goodwill that would be Included in the February 1, 2014, financial statement of Arlington.

a. P200,000 b. P230,000 c. P100,000 d. P130,000

Answer: C.
Cost of acquiring Winston
Cash P1,400,000
Shares of stocks ( 10,000 x 80) 800,000 2,200,000
Fair value of net assets acquired:
Inventory P600,000
Land 500,000
Building 1,000,000 (2,100,000)
Goodwill P100,000

B. Assume that Arlington pays cash of P20 million. No stock is issued. An additional 40,000 Is paid In direct
combination costs, determine the net gain from business combination.

a. P100,000 b, P200,000 c. 260,000 d. 60,000

Answer: D.
Gain from business combination must be P60,000.
Cost of acquiring Winston P2,000,000
Fair value of net assets acquired 2,100,000
Additional Cost (40,000)
Net gain from business combination P60,000
7. On December 31, 2015, Seco Company paid P 950,000 for 95% of the outstanding common stock of
Sana Company. The remaining 5% was held by a stockholder who was unwilling to sell the stock. Sana's
net assets had a book value of P 810,000 and a fair market value of P 900,000 when it was acquired by
Seco. If Sana uses push- down accounting, the non- controlling interest should be reported at:

a. P 40,500 b. P 50,000 c. P 47,500 d. P 45,000

Answer: b. P 50,000

Solution:

Acquisition Cost P 950,000


Divided by: 95%
Total P 1,000,000
Multiplied by: 5%
Non- controlling Interest P 50,000

8. Ambrose Company acquires a controlling interest in Monica Company in the open market for P 220,0
00. The P 200 par value capital stock of Monica Company at the date of acquisition is P 250,000 and its
retained earnings amounts to P 100,000. The market value per share of Monica Company is P 220 per
share. In the consolidated statement of financial position on the date of acquisition, non- controlling
interest would show a balance of:

a. P 55,000 b. P 60,000 c. P 62,500 d. P 50,000

Answer: a. P 55,000

Solution:

Controlling (Parent) Interest:


Shares Acquired ( P 220,000/ P 220) 1,000 shares
Divided by Shares Outstanding ( P 250,000/ P 200)/ 1,250
Parent's Interest 80%
P 220,000/ 80% = P 275,000
Non- controlling/ Minority Interest in Net Assets of Subsidiary:
( P 275,000 x 20% ) = P 55,000

9. On August 31, 2016, Company P acquires 75% (750,000 ordinary shares) of Company S for P7,500,000
(P10 per share). In the period around the acquisition date, Company S's shares are trading at about P8 per
share. Company P pays premium over market because of the synergies it believes it will get. It is therefore
reasonable to conclude that the fair value of Company S as a whole may not be P10,000,000. In fact, an
independent valuation shows that the value of Company S is P9,700,000 ( fair value of Company S).
Assuming that the fair value of the net identifiable assets is P8,000,000 (carrying value is P6,000,000)

Goodwill arisung on consolidation is to be valued on the proportionate basis or "Partial" Goodwill:

a. P200,000
b. P1,500,000
c. P1,700,000
d. P2,000,000

Answer: B

Fair value of subsidiary:

Consideration transferred:. P7,500,000


Less:book value of Net assets
(P6,000,000 x 75%). 4,500,000
Allocates excess. P3,000,000
Less: over/under valuation of Assets and
Liabilities ((P8,000,000 - P6,000,000) x75% 1,500,000

Goodwill(partial). P1,500,000

10. Mark, a private limited company, has arranged filorman, a public limited company, to acquire it as a
means of obtaining a stock exchange listing. Man issue 15 million shares acquire the whole of the share
capital of Mask (6 million shares). The fair value of the net assets of Mask and Man are P30 million and P18
million respectively. The fair value of each of the share of Mask is P6 and the quoted market price of Man's
share is P2. The share capital of Man is P25 million shares of acquisition. Compute the value of goodwill in
the above acquisition.

a. P16 million
b. P12 million
c. P 6 million
d. P10 million

Answer: C

Consideration transferred (4,000,000 shares* x P6) P24,000,000


Less: book value of equity —Man .
(P18,000,000 x 100%). 18,000,000
Allocated excess P 6,000,000
Less: over/under valuation of assets and
liabilities( book value sme fair value). 0

Goodwill P 6,000,000

100%
Man – – – – – > Mask
Currently issued . 15M 60%** 6M 60%
Additional shares issued. 10M 40% <—– 4M /40%

Total shares. 25M. 10M

11. Condensed Statement of Financial Position of Dolce Inc. and Gabbana Inc. as of 12/31/2011 were as
follows:
Dolce Gabbana
Current assets 275,000 P65,000
Noncurrent 625,000 425,000
assets
Total assets 900,000 490,000
Liabilities 65,000 35,000
549,700 296,700
Ordinary
shares, P23 Par

Share Premium 35,300 28,300


Accumulated 250,000 130,000
Profits (losses)

On January 1, 2012, Dolce Inc. issued 30,000 shares with market value of P25/share for the assets and
liabilities of Gabbana Inc. Dolce Inc. also paid P125,000 cash. The book value reflects the fair value of the
assets and liabilities, except that the non-current assets of Gabbana Inc. have fair value of P630,000 and
the noncurrent assets of Dolce Inc. are overstated by P30,000. Contingent consideration, which is
determinable, is equal to P15,000. Dolce paid for the share issuance costs only amounting to P74,000 and
incurred other acquisition costs amounting to P19,000.

As a result of acquiring the net assets of Gabbana Inc., compute for the total liabilities in the books of
Dolce.

a. P 100,000 b. P 115,000 c. P 134,000 d. P 65,000

Ans. C
Liabilities of Dolce P65,000
Liabilities of Gabbana P35,000
Contingent Consideration P15,000
Acquisition Cost Incurred P19,000
Total liabilities P134,000

12. On Dec. 31,2013, P Inc. paid P495,000 cash for all the outstanding stock of S Company. S’s assets and
liabilities on that day were as follows:

Cash P60,000

Inventory 150,000

P.P.E (net of accumulated dep. of P100,000) 350,000

Liabilities 70,000

On the day of business combination the fair value of the inventory was P125,000 and the fair value
of P.E (net) was P385,000. The goodwill (income from acquisition) resulting from this acquisition
amounts to:

a. (P5,000)

b. P85,000

c. P40,000

d. P5,000

Ans. A

Acquisition cost P495,000


Less: Book value of interest acquired (P560,000 – P70,000) 490,000
Difference 5,000
Allocation:
Inventory P 25,000
Property and equipment ( 35,000) (10,000)
Income from acquisition P( 5,000)

13. The balance sheets of Pedro Ltd. and Santi Ltd. on June 30, 2016 were as follows:

Pedro Ltd Santi Ltd


Current assets P500 P700
Non-current assets 1, 300 3, 000
Total assets P1, 800 P3, 700
Share capital:
100 shares P300
60 shares P600
Retained earnings 800 1, 400
P1, 100 P2, 000
Current liabilities P300 P600
Non-current liabilities 400 1, 100
P700 P1, 700
Total equity and liabilities P1, 800 P3, 700

On July 1, Pedro Ltd. acquired all the issued shares of Santi Ltd. giving in exchange 2 ½ Pedro Ltd. shares
for each ordinary share of Santi Ltd. Pedro Ltd, thus issued 150 shares to acquire the 60 shares issued by
Santi Ltd.

The fair value of each ordinary share of Santi Ltd. on July 1, 2016 is P40, while the quoted market price of
Pedro Ltd.’s ordinary shares is P16. The fair values of Pedro Ltd.’s identifiable assets and liabilities at
acquisition date are the same as their carrying amounts except for the non-current assets whose fair value
was P1, 500. The tax rate is 30%.

The amount of goodwill acquired on July 1, 2016:


a. P1, 160
b. P856
c. PP400
d. P360

Ans. : D
Solution:
Consideration transferred (40 shares* x P40) P1, 600
Less: Book value of SHE – Pedro Ltd. (P300 + P800) x 100% 1,100
Allocated excess P500
Less: Over/under valuation of Assets and Liabilities:
Increase in Non-current assets: [(P1,500 – P1,300) x 100% x 70%] 140
Goodwill P360

100%
Pedro Ltd. Santi Ltd.
Currently issued 150 60%** 60 60%
Additional shares issued 100 40% 40 40%
Total shares 250 100

**150/250

Pedro Ltd, issued 2 ½ shared in exchange for each ordinary share of Santi Ltd. All of Santi Ltd.’s
shareholders exchange their shares for Pedro Ltd. Pedro Ltd. Therefore issues 150 shares (60 x 2 ½) for
the 60 shares in Santi Ltd.

Pedro Ltd. is now the legal parent of the subsidiary, Santi Ltd. However, analyzing the shareholding in
Pedro Ltd. shows that it consists of the 100 shares existing prior to the merger and 150 new shares held by
former shareholders in Santi Ltd. In essence, the former shareholders of Santi Ltd. now control both
entities Pedro Ltd. and Santi Ltd. The former Santi Ltd. shareholders have a 60% interest in Pedro Ltd
[150/(100 + 150)]. The IASB argues that there has been a reverse acquisition, and that Santi Ltd. is
effectively the acquirer of Pedro Ltd.

The key accounting effect of deciding that Santi Ltd. is the acquirer is that the assets and liabilities of Pedro
Ltd. are to be valued at fair value. This is contrary to normal acquisition accounting, based on Pedro Ltd.
being the legal parent of Santi Ltd., which would require the assets and liabilities of Santo Ltd. to be valued
at fair value.

14. Mask, a private limited company, has arranged for Man, a public limited company, to acquire it as a
means of obtaining a stock exchange listing. Man issues 15 million shares to acquire the whole of the
share capital of Mask (6 million shares). The fair value of the net assets of Mask and Man are P30 million
and P18 million respectively. The fair value of each of the shares of Mask is P6 and the quoted market
price of Man’s share is P2. The share capital of Man is 25 million shares after the acquisition. Calculate the
value of goodwill in the above acquisition.

a. P16 million
b. P12 million
c. P10 million
d. P6 million

Ans. : D
Solution:

Consideration transferred (4, 000, 000 shares x P6) P24, 000, 000
Less: Book value of SHE – Man: P18, 000, 000 x 100% 18, 000, 000
Allocated excess P6, 000, 000
Less: Over/Under valuation of Assets and Liabilities
(book value same fair value) 0
Goodwill P6, 000, 000

15. Clarisse Company acquires a controlling interest in Mimi Company in the open market for P120,000.
The P100 par value capital stock of Mimi Company at the date of acquisition is P125,000 and it’s retained
earnings amounts to P50,000. The market value per share of Mimi Company is P120 per share. In the
consolidated statement of financial position on the date of acquisition. Non controlling interest would show
a balance of:

a. P30,000
b. P40,000
c. P25,000
d. P17

Ans. A

Controlling (Parent) interest:


Shares acquired (P120,000 / P120) 1,000 shares
Divided shares outstanding (P125,000 /P100) ÷1,250
Parent’s interest 80%
Minority interest in net assets of subsidiary (P150,000 x 20%) P30,000

16. On the day of acquisition Anne Inc. had the following assets and liabilities:

Book Value Fair Value


Current assets P100,000 P100,000
Plant assets(net) 220,000 260,000
Liabilities) (40,000) (40,000)
Sean Company paid P450,000 for 90% of the outstanding voting stock of Anne. The goodwill in the
consolidated statement of financial position at acquisition is:
P190,000
P120,000
P180,000
P230,000

Ans. C

Parent Company Interest P450,000


NCI 50,000
Consideration P500,000
Less: Fair value of net assets 320,000
Goodwill P180,000

17. Seminarian. Inc. has 100,000 shares of P2 par value stock outstanding. Priests Corporation acquired
30,000 shares of Seminarian’s shares on January 1, 20x4 for Pl20000 when Seminarian’s net assets had a
total fair value of P350000. On July 1, 20x7, Priests agreed to buy an additional 60,000 shares of
Seminarian from single stockholder for P6 per share“ Although Seminarian’s share 5. were selling in the P5
range around July 1, 20x7. Priests forecasted that obtaining control of Seminarian would produce
significant revenue synergies to justify the premium price paid. if Seminarian‘s net identifiable assets had a
fair value of P500000 on July i, 20x7, how much goodwill on full fair value basis should Priests report in its,
post-combination consolidated balance sheet?

A. P -0-
B. P 60,000
C. P 90,000
D. P 100,000

ANSWER: B

60% FV, stocks issued: 60,000 shares x P6, fair value P 360,000
30% FV, of previously held equity interest: 30,000 shares x P5 fair
150,000
value

10% FV of NCI (100,000-60,000-30,000) x P4, fair value 40,000


100% FV of subsidiary P 560,000
Less: fair value of net asset of subsidiary 500,000

19. Robin Corporation purchased 150,000 previously unissued shares of Nest Company's $10 par value
common stock directly from Nest tor $3,400,000. Nest's stockholder's equity immediately before the
investment by Robin consisted of $3,000,000 of capital stock and $2,600,000 in retained earnings. What
is the book
value of Robin's investment in Nest?

a. $1,500,000.
b. $1,680,000.
c. $2,800,000.
d. $3,000,000.

Answer: d

Shares outstanding before new shares are issued $ 300,000


Shares issued to Robin $ 150,000
Total shares outstanding $ 450,000
Percentage owned by Robin equals 150,000/450,000= 33.33%

Stockholders' equity before new shares are issued $ 5,600,000


+lnvestment by Robin $ 3,400,000
=Stockholders' equity after Robin investment $ 9,000,000
x Robin's percentage ownership 33.33%
=Book value of Robin's interest $ 3,000,000

20. Pogi Corporation paid P 100,000 cash for the net assets of Ganda Corporation which consisted of the
following:

Book Value Fair Value


Current Assets P 98,000 P 120,000
Property and Equipment P 350,000 P 400,000
Liabilities assumed P 100,000 P 110,000

The property and equipment acquired in this business combination should be recorded at what amount?

a. P 100,000
b. P 80,000
c. P 350,000
d. P 400,000

Answer: d

The Property and equipment should be recorded at its Fair Value of P 400,000

21. Ice, a private limited company, has arranged for Cream, a public limited company, to acquire it as a
means of obtaining a stock exchange listing. Cream issues 15 million shares to acquire the whole of the
share capital of Ica (6 million shares). The fair value of the net assets of Ice and Cream are P30 million and
P18 million respectively. The fair value of each of the shares of Ice is P6 and the quoted market price of
Cream’s shares is P2. The share capital of Man is 25 million shares after the acquisition. Calculate the
value of goodwill in the above acquisition.
a. P16 million c. P10 million
b. P12 million d. P 6 million

Ans: d

Consideration transferred (4 million shares* x P6) P24,000,000


Less: BV of SHE – Cream: P18 million x 100% 18,000,000
Allocated excess P 6,000,000
Less: Over/Under valuation of Assets and Liabilities
(book value same fair value) 0
Goodwill P 6,000,000

22. Red, Inc. has 100,000 shares of P2 par value stock outstanding. Velvet Corporation acquired 30,000
shares of Red’s shares on January 1, 20x5 for P120,000 when Red’s net assets had a total fair value of
P350,000. On July 1, 20x8, Velvet agreed to buy an additional 60,000 shares od Red from single
stockholder for P6 per share. Although Red’s shares were selling in the P5 range around July 1, 20x8,
Velvet forecasted that obtaining control of Red would produce significant revenue synergies to justify the
premium price paid. If Red’s net identifiable assets had a fair value of P500,000 on July 1, 20x8, how much
goodwill on full fair value basis should Velvet report in its post-combination consolidated balance sheet?
a. P 0 c. P 90,000
b. P60,000 d. P100,000

Ans: b

(60%) Fair value of consideration given


Shares: 60,000 shares x P6, fair value P 360,000
(30%) Fair value on previously held equity interest
30,000 shares x P5, fair value 150,000
(10%) Fair value of non-controlling interest
(100,000 – 60,000 – 30,000) x P5, fair value 50,000
(100%) Fair value of Subsidiary P 560,000
Less: Fair value of Net Assets (SHE of Subsidiary) 500,000
Goodwill (Full/Gross-up) P 60,000

23. Aquino Corp. acquired all the assets and liabilities of Binay Corp. by issuing shares of its common stock
on January 1, 2016. Partial balance sheet data for the companies prior to the business combination and
immediately following the combination is provided:

Aquino Binay

Book Book Combinatio


Value Value n

Cash………………………………………. P 130,000 P 50,000 P 180,000

Accounts receivable…………………….. 144,000 40,000 188,000

Inventory………………………………….. 66,000 90,000 176,000

Plant and equipment, net……………….. 800,000 300,000 1,300,000

Goodwill…………………………………… ?

Total Assets………………………………. P1,140,00 P480,000 P ?


0

Accounts P P 50,000 P 150,000


payable………………………… 100,000
Bonds payable……………………………. 500,000 200,000 700,000

Common stock, P2 par………………….. 200,000 50,000 320,000

Additional paid-in capital………………… 130,000 40,000 490,000

Retained earnings……………………….. 210,000 140,000 ?

Total Liabilities and Equities………......... P1,140,00 P480,000 P ?


0

What number of shares and in what price did Aquino issue for this acquisition, as well as the
amount of goodwill to be reported by the combined entity immediately following the combination?
a. P80,000; P8; P450,000
b. P60,000; P8; P454,000
c. P40,000; P6; P456,000
d. P20,000; P6; P460,000
ANSWER: (b)
Common stock – combined………………………… P320,000

Common – Acquirer Aquino……………………….. 200,000

Common stock issued……………………………… P120,000

Divided by: Par value of common stock………….. P 2

Number of Aquino shares to acquire Binay……… 60,000 (b


)

Paid-in capital books of Aquino (P200,000 + P330,000


P130,000)…

Paid-in capital in the combined balance sheet

(P320,000 + 810,000
P490,000)………………………………..
Paid in capital from the shares issued to acquire P480,000
Binay….

Divided by the number of 60,000


shares……………………………
Fair value per shares when sock is P 8 (
issued……….............. b
)

Net identifiable assets of Aquino before acquisition:

(P130,000 + 144,000 + 66,000 + 800,000 -


100,000 – 500,000)…………………………........... P540,000

Net identifiable assets in the combined balance sheet:

(P180,000 + 188,000 + 176,000 + 1,300,000 -

150,000 – 700,000………………………………….. 994,000

Fair value of the net identifiable assets held by Binay

at the date of acquisition………………………………… P454,000 (


b
)

24. Richard, Inc. is to acquire Raymond Corp. by absorbing all the assets and assuming all the liabilities to
the latter in exchange for shares of the former’s stock. Below are the balance sheets of the two companies
with the corresponding appraised value increment for Raymond Corp.
Richard Raymond

Assets, per books………………………………… P2,000,000 P1,250,000

Assets, appraised increase……………………… 150,000

Liabilities…………………………………………… P 750,000 P 400,000

Common stock (No par; P100 par)……………… 1,000,000 500,000

Additional paid-in capital…………………………. 350,000 150,000

Retained earnings (deficit)………………………. (100,000) 200,000

Total Equities……………………………………… P2,000,000 P1,250,000

The parties agree to use the appraised values, against which the fair market value of the shares will
be matched. Richard, Inc.’s common stock is currently selling at P100 per share. The number of
share to be issued by Richard, Inc. is:
a. 20,000
b. 15,000
c. 13,000
d. 10,000
ANSWER: (d)
Assets at appraised value (P1,250,000 + P150,000)………………… P1,400,000
Less: Liabilities……………………………………………………………. 400,000
Net assets at appraised values…………………………………………. P1,000,000
Divided by: Current selling price per share…………………………….. 100

Number of shares issued…………………………………………………. 10,000

25. On July 1, 2016, Parent Ltd. acquired all the issued share capital of Sub Ltd. giving in exchange of
100,000 shares in Parent Ltd. these having a fair value of P5 per share. At acquisition date, the balance
sheets of Parent Ltd. and Sub Ltd. and the fair values of Sub Ltd's assets and liabilities, were as follows:

Parent Ltd. Sub Ltd.


Carrying Amount Carrying Amount Fair Value
EQUITY AND LIABILITIES
Equity

Share Capital P550,000 P300,000


Retained earnings 350,000 140,000
Total equity P900,000 P440,000
Liabilities
Provisions P 30,000 P 60,000 P 60,000
Payables. 27,000. 34,000. 34,000
Tax liabilities 10,000 6,000 6,000
Total liabilities P 67,000 P100,000
Total equity and liabilities P967,000 P540,000

ASSETS
Land P120,000 P150,000 P170,000
Equipment 620,000 480,000 330,000
Accumulated depreciation (180,000) (170,000)
Investment in subsidiary 500,000
Inventory. 92,000 75,000 80,000
Cash 15,000 5,000 5,000
Total Assets P967,000 P540,000

At acquisition date, Sub Ltd.has an unrecorded patent with a fair value of P20,000 and a contingent liability
of with a fair value of P15,000. The tax rate is 30%.

The amount of goodwill acquired on July 1, 2016:

a. P25,000
b. P15,000
c. P10,000
d. Zero

ANSWER: A

Consideration transferred (100,000 shares x P5) P500,000


Less. Book value of SHE - S: P440,000 x 440,000
100℅ Allocated excess P 60,000
Less: Over/Under validation of Assets and Liabilities
Increase in Land: P20,000 x 100% x 70% P14,000
Increase in Equipment: P20,000 x 100% x 70% 14,000
Increase in Inventory: P5,000 x 100% x 70% 3,500
Increase in Parent: P20,000 x 100% x 70% 14,000
Increase in Provision: P(15,000) x 100% x 70% (10,500) 35,000
Goodwill P 25,000

26. Oh January 1, 2016, Park Corporation and Stand Corporation and their condenser balance sheet
are as follows:
Park Corp. Strand Corp.
Current Assets P 70,000 P 20,000
Non-current Assets 90,000 40,000
Total Assets P160,000 P 60,000
Current Liabilities P30,000 10,000
Long-term Debt 50,000 -
Stockholders' Equity 80,000 50,000
Total Liabilities and Equities P160,000 P60,000

On January 2, 2016, Park Corporation borrowed P60,000 and used the proceeds to obtain 80% of the
outstanding common shares of Strand Corporation. The acquisition price was considered proportionate to
Stand's fair value.

The P60,000 debt is payable in 10 equal annual principals, plus interest, beginning December 31, 2016.
The excess fair value of the investment over the underlying book value of the acquired net assets is
allocated to inventory (60%) and to goodwill (40%).

On the consolidated balance sheet as of January 2, 2016, what should be the amount of each of goodwill
using proportionate basis (partial)?

a. P 0
b. P8,000
c. P10,000
d. P20,000

ANSWER: B

Fair Value of consideration given P60,000


Less: Book value of net assets (P50,000 x 80%) 40,000
Allocated excess P20,000
Less: Over/Under validation of Assets and Liabilities
Increase in Inventory (P20,000 x 60% = P12,000 / 80%
= P15,000 increase in inventory x 80% 12,000
Goodwill (partial). P 8,000

27. On June 12, 2015 Don Company purchases 8,000 shares of Sam Company for P68 per share.
Just prior to the purchase, Sam Co. has the following statement of financial position.

ASSETS Liabilities & Equity


Cash P 20,000 Current liabilities P 250,000
Inventory 280,000 Common Stock, P5 par 50,000
Equipment 400,000 APIC 130,000
Goodwill 100,000 Retained Earnings 370,000
Total Assets P 800,000 Total Liabilities and EquityP 800,000

On June 12,2015 Sam’s inventory has a fair value of P450,000 and that the equipment is worth
P600,000. What is the amount of non controlling interest in the consolidated statement of financial
position on the date of acquisition?
a. P128,000
b. P150,000
c. P164,000
d. P120,000

ANSWER: C
Rationale
Cash P 20,000
Inventory 450,000
Equipment 600,000
Total Assets 1,070,000
Liablities (250,000)
Net Assets P820,000
X 20%
NCI P164,000

28. Using the data in the preceding number what is the amount of goodwill (gain on acquisition) to
be reported in the consolidated statement of financial position on the date of acquisition?
a. P98,000
b. P100,000
c. P112,000
d. P106,000

ANSWER: C
Rationale
Acquisition Cost (8000 x 68) P544,000
NCI 164,000
Total P708,000
Less: Net Assets P820,000
Gain on Acquisition (P112,000)

29. The balance sheets of Min Ltd. and Kim Ltd on June 30,2017 were as follows:
Min Ltd Kim Ltd
Current assets P 600 P 800
Non-current assets 1 200 2 900
Total assets P1 800 P3 700
Current Liabilities P 250 P 700
Non-current liabilities 450 1 000
P700 P 1 700
Share Capital
100 shares P 300
60 shares P 600
Retained Earnings 800 1 400
P1 100 P2 000
Total equity and liabilities P1 800 P3 700

On July , 2017, Min Ltd acquired all the issued shares of Kim Ltd giving in exchange 21/2 Min Ltd shares for
each ordinary share of Kim Ltd. Min Ltd thus issued 150 shares to acquire the 60 shares issued by Kim Ltd.

The fair value of each ordinary share of Kim Ltd on July 1,2017is P40, while the quoted market price of Min
Ltd’s ordinary shares is P16. The fair values of Min Ltd’s identifiable assets and liabilities at acquisition date
are the same as their carrying amounts except for the non-current assets whose fair value was P1 500.
The tax rate is 30%.

The amount of goodwill acquired on July 1,2016 is


a. P1 160 c. P400
b. P 856 d. P360
Solution:
ANS: D
Consideration transferred (40 shares* x P40) P1 600
Less: Book value of SHE-Pedro Ltd (P300 + P800) x 100% 1 100
Allocated Excess P 500
Less: Over/Under valuation of Assets and Liabilities: Increase
in Non-current assets: [(P1 500 – P1 300) x
100% x 70% 140
Goodwill P 360

100%
Min Ltd Kim Ltd
Currently issued 150 60% 60 60%
Additional shares issued 100 40% 40 / 40%
Total shares 250 100

**150/250

Min Ltd issues 21/2 shares in exchange for each ordinary share of Kim Ltd. all of Min Ltd’s shareholders
exchange their shares for Min Ltd. Min Ltd therefore issues 150 shares (60 x 21/2) for the 60 shares in Kim
Ltd.

Min Ltd is now the legal parent of the subsidiary, Kim Ltd. however, analyzing the shareholding in Min Ltd
shows that it consists of the 100 shares existing prior to the merger and 150 new shares held by former
shareholders in Kim Ltd. In essence, the former shareholders of Kim Ltd now control both entities Min Ltd
and Kim Ltd. The former Kim Ltd shareholders have a 60% interest in Min Ltd[150/(100+150)]. The IASB
argues that there has been a reverse acquisition, and that Kim Ltd is effectively the acquirer of Min Ltd.

30. On July 1,2016, Naly Co. Acquired all the issued share capital of Lito Co. giving in exchange of 120,000
shares in Naly Co. these having a fair value of P5 per share. At acquisition date, the balance sheets of Naly
Co. And Lito Co. And the fair values of Lito Co.’s assets and liabilities were as follows:
Naly Co. Lito Co.
Carrying Amount Carrying Amount Fair Value
ASSETS
Cash P 15,000 P 5,000 P 5,000
Inventory 92,000 75,000 80,000
Investment in Subsidiary
(Shares in Lito Co.) 500,000
Equipment 620,000 480,000 330,000
Accumulated depreciation (180,000) (170,000)
Land 120,000 150,000 170,000
TOTAL ASSETS P967,000 P540,000

LIABILITIES AND EQUITY


Liabilities
Payables P 27,000 P34,000 P34,000
Tax liabilities 10,000 6,000 6,000
Provisions 30,000
Total liabilities P 67,000 P 40,000
Equity
Share capital P550,000 P360,000
Retained earnings 350,000 140,000
Total equity P900,000 P500,000
TOTAL EQUITY AND LIABILITIES P967,000 P540,000

At acquisition date, Lito Co. Has an unrecorded patent with a fair value of P20,000 and a contingent liability
with a fair value of P15,000. The tax rate is 30%.

The amount of goodwill acquired on July 1,2016:


a. P65,000 c. P50,000
c. P45,000 d. Zero
Solution:
ANS: A
Consideration transferred (120,000 shares x P5) P600,000
Less: Book value of SHE-Lito P500,000 x 100% 500,000
Allocated excess P100,000
Less: Over/Under valuation of Assets and Liabilities:
Increase in Land: P20,000 x 100% x 70% P14,000
Increase in Equipment: P20,000 x 100% x 70% 14,000
Increase in Inventory: P5,000 x 100% x 70% 3,500
Increase in Patent: P20,000 x 100% x 70% 14,000
Decrease in Provision : (P20,000) x 100% x 70% ( 10,500) 35,000
Goodwill P 65,000

The net fair value of the subsidiary could be calculated by revaluing the assets and liabilities of the
subsidiary from the carrying amounts to fair value, remembering that under PAS No. 12 Income Taxes
revaluation of assets requires a recognition of the tax effect of the revaluation because there is a
difference between the carrying amount and the tax base caused by the revaluation.

31. On January 1, 2016, Park Corporation and Strand Corporation and their condensed balance
sheet are as follows:

Park Corporation Strand Corporation

Current Assets……………………………….. P70,000 P20,000

Non-current 90,000 40,000


Assets…………………………..
Total Assets………………………………….. P160,000 P60,000
Current P30,000 P10,000
Liabilities…………………………….
Long term 50,000 -
debt……………………………….
Stockholder’s 80,000 50,000
Equity…………………………
Total Liabilities and P160,000 P60,000
Equities………………..

On January 2, 2016, Park Corporation borrowed P60,000 and used the proceeds to obtain 80% of
the outstanding common shares of Strand Corporation. The acquisition price was considered
proportionate to Strand’s fair value. The P60,000 debt is payable in 10 equal annual principal
payments, plus interest, beginning December 31,2016. The excess fair value of the acquired net
assets is allocated to inventory (60%) and to goodwill (40%).

On a consolidated balance sheet as of January 2, 2016, what should be the amount of goodwill
using proportionate basis (partial)?

a. P0
b. P8,000
c. P10,000
d. P20,000
Answer: B

Fair value of consideration given P60,000

Less: Book value of net assets (P50,000 x 80%) 40,000

Allocated excess P20,000

Less: Increase in inventory (P20,000x60%)/80%

=P15,000 increase in inventory x 80% 12,000

Goodwill (partial) P8,000

32. Pita Company acquires a controlling interest in Soda Company in the open market for
P120,000. The P100 par value capital stock of Soda Company at the date of acquisition is
P100,000 and its retained earnings amounts to P50,000. The market value per share of Soda
Company is P150 per share. In the consolidated statement of financial position, non-controlling
interest would show a balance of:

a. P80,000
b. P5,000
c. P30,000
d. P35,000
Answer: C

Controlling (Parent) interest:

Shares acquired (P120,000 / P150) 800 shares

Divided shares outstanding (P100,000 /P100) ÷1,000

Parent’s interest 80%

Minority interest in net assets of subsidiary (P120,000 / 80% x 20%) P30,000

33. AA Company acquired all the issued share capital of BB Company on March 31,2016 giving in
exchange of 100,000 shares in AA Co. These having a fair value of P5 per share. At acquisition date, the
balance sheets of AA Co and BB Co. And fair values of BB Co.’s assets and liabilities,were as follows:

AA Co. BB Co.

Carrying Amount Carrying Amount Fair Value

Assets:

Cash........................................................................P 15,000 P 5,000 P 5,000

Inventory................................................................. 92,000 75,000 80,000

Investment in Subsidiary

(Shares in BB Co.) ............................................. 500,000

Equipment ............................................................ 620,000 480,000 330,000

Accumulated depreciation .................................. (180,000) (170,000)

Land ................................................................... 120,000 150,000 170,000

Total Assets.................................................. P 967,000 P 540,000

Liabilities and Equities:

Liabilities

Tax Liabilities .................................................. P 10,000 P 6,000 P 6,000


Payables ............................................................27,000 34,000 34,000
Provisions ..................................................... 30,000 60,000 60,000

Total Liabilities............................................... P 67,000 P100,000


Equity
Share Capital ............................................. P 550,000 P300,000
Retained Earnings ..................................... 350,000 140,000
P 900,000 P440,000

Total Liabilities and Equity ........................... P 967,000 P540,000

At acquisition date, BB Co. Has an unrecorded patent with a fair value of P20,000 ,contingent liability with
a fair value of P15,000, and one of the payables is a dividend payable of P8,000. AA Co. acquires the
shares of BB Co. On a cum div basis or “dividends-on” arrangement.The tax rate is 30%. The amount of
goodwill acquired on March 31,2016:

A.P 16,000
B. P 17,000
C.P 18,000
D.P 19,000

Answer: B

Consideration transferred[(100,000 shares x P5) - P8,000] ................................................ P 492,000


Less:Book Value of Shareholders Equity-BB Co:
P440,000 x 100% ....................................................................................................... 440,000

Allocated Excess................................................................................................................. P 52,000


Less: Over/Under valuation of Assets and Liabilities:
Increase in Land: P20,000 x 100% x 70% .......................... P 14,000
Increase in Equipment: P20,000 x 100% x 70% .................. 14,000
Increase in Inventory: P5,000 x 100% x 70% .........................3,500
Increase in Patent:P20,000 x 100% x 70% ........................... 14,000
Increase in Provision: P(15,000) x 100% x 70%.................(10,500) 35,000
Goodwill ........................................................................................................................... P 17,000 (B)

34. What would be the effect on the consolidated financial statements if an unconsolidated subsidiary is
accounted for by the cost method of accounting, but consolidated financial statements are prepared for
other subsidiaries?

a. All the unconsolidated subsidiary's ledger account balances would be included individually in the
consolidated financial statements.
b. Consolidated net income would not include any amounts for the unconsolidated subsidiary.

c. Consolidated net income would be the same as if the subsidiary had been included in the consolidation.

d. Dividend revenue from the unconsolidated subsidiary would be included in consolidated net income.

Answer: D

35. Under SFAS 141R, what value of the assets and liabilities is reflected in the financial statements on the
acquisition date of a business combination?

a. Carrying value
b. Fair value
c. Book value
d. Average value

Answer: b

36. The parent company owned 65% of subsidiary’s net assets. On the consolidated statement of financial
position of the combined entity, the retained earnings has amount equal to:

a. The subsidiary’s retained earnings.

b. The 65% of the subsidiary’s retained earnings plus parent company’s retained earnings.

c. The 100% of the subsidiary’s retained earnings plus parent company’s retained earnings.

d. The parent company’s retained earnings.

ANSWER: (d)

Only the parent company’s retained earnings will appear on the consolidated statement of financial
position.

37. Which is not true about the working paper for consolidated statement of financial position on the date
of acquisition?
a. The amounts in the consolidated column reflects the financial position of single economic entity
comprising two legal entities, after eliminating all intercompany balances
b. Consolidated retained earnings include only the retained earnings of the parent company
c. The elimination entry is recorded in the parent and subsidiary’s accounting records
d. The consolidated paid-in capital amounts are those of the parent company only.
Ans. C

38. On December 31,2013, Palo Company paid P990,000 for 99% of the outstanding coomon stock of Sota
Company. The remaining 1% was held by a stockholder who was unwilling to sell the stock. Sota’s net assets
had a book value of P850,000 and a fair market value of P900,000 when it was acquired by Palo. If Sota
uses push-down accounting, the non-controlling interest should be reported at:
a. P8,500
b. P9,000
c.P9,900
d. P10,000
Ans. B
Solution:
P900,000 x 1%= P9000
39. Rizal Corporation paid P 100,000 cash for the net assets of Bonifacio Corporation which consisted of
the following:

Book Value Fair Value


Current Assets P 98,000 P 120,000
Property and Equipment P 350,000 P 400,000
Liabilities assumed P 100,000 P 110,000

The property and equipment acquired in this business combination should be recorded at what amount?

a. P 400,000
b. P 80,000
c. P 350,000
d. P 100,000

Answer: A

40. The Property and equipment should be recorded at its Fair Value of P 400,000
On January 2, 2011, Bulalo Co. purchased 75% of Pares Co’s outstanding common stock. On that date, the
fair value of the 25% noncontrolling interest was P35,000. During 2011, Pares had net income of P20,000.
Selected balance sheet data at December 31,2011, is as follows:

Bulalo Pares

Total assets P420,000 P180,000

Liabilities P120,000 P60,000

Common stock 100,000 50,000

Retained Earnings 200,000 70,000

During 2011 Bulalo and Pares paid cash dividends of P25,000 and P5,000 respectively, to their
shareholders. There were no other intercompany transactions.

In Bulalo’s December 31,2011 consolidated balance sheet, what amount should be reported as
noncontrolling interest in net assets?

a. P30,000

b. P35,000

c. P38,750

d. P40,000
ANSWER: C

Fair value of noncontrolling interest P35,000

Plus: Share of net income (25% x 20,000) 5,000

Less: Share of dividends (25% x 5,000) (1,250)

Noncontrolling interest P38,750

You might also like